The width of a rectangular parking lot is 53 ft less than its length. Determine the dimensions of the parking lot if it measures 250 ft diagonally.

Answers

Answer 1

Answer:

The dimension of the lot is 201.28 ft by 148.28 ft

Step-by-step explanation:

Given;

diagonal of the parking lot, d = 250 ft

let the length of the parking lot = L

the width, W = L - 53

The diagonal of the lot, length of the lot, and width of the lot form a right triangle.

Apply Pythagoras theorem to determine the length, L

L²  +  W² = 250²

L²  + (L - 53)² = 250²

L²  +  L²  - 106L  +  2809 = 62,500

2L²  -  106L  + 2809  -  62,500 = 0

2L²  -  106L  - 59,691 = 0

this forms quadratic equation; a = 2, b = -106 and c = -59,691

[tex]L = \frac{- b \ \ +/- \ \ \sqrt{b^2 - 4ac} }{2a} \\\\L = \frac{- (-106) \ \ +/- \ \ \sqrt{(-106)^2 - 4(-59691 \times 2)} }{2(2)}\\\\L = \frac{106 \ \ +/- \ \ \sqrt{488764} }{4} \\\\L = \frac{106 + 699.12}{4} \\\\L = 201.28 \ ft[/tex]

The width, W = 201.28  -  53

                  W = 148.28 ft


Related Questions


Which point on the x-axis lies on the line that passes point C and is parallel to line AB

Answers

Answer:

(2,0)

Step-by-step explanation:

In how many ways can 10 people be seated in 10badjacenr seats in a theater​

Answers

3,628,800 is the correct answer

all the cubes root of
[tex] \sqrt{3 + i} [/tex]

Answers

If you're looking for the cube roots of √(3 + i ), you first have to decide what you mean by the square root √(…), since 3 + i is complex and therefore √(3 + i ) is multi-valued. There are 2 choices, but I'll stick with 1 of them.

First write 3 + i in polar form:

3 + i = √(3² + 1²) exp(i arctan(1/3)) = √10 exp(i arctan(1/3))

Then the 2 possible square roots are

• √(3 + i ) = ∜10 exp(i arctan(1/3)/2)

• √(3 + i ) = ∜10 exp(i (arctan(1/3)/2 + π))

and I'll take the one with the smaller argument,

√(3 + i ) = ∜10 exp(i arctan(1/3)/2)

Then the 3 cube roots of √(3 + i ) are

• ∛(√(3 + i )) = ¹²√10 exp(i arctan(1/3)/6)

• ∛(√(3 + i )) = ¹²√10 exp(i (arctan(1/3)/6 + π/3))

• ∛(√(3 + i )) = ¹²√10 exp(i (arctan(1/3)/6 + 2π/3))

On the off-chance you meant to ask about the cube roots of 3 + i, and not √(3 + i ), then these would be

• ∛(3 + i ) = ⁶√10 exp(i arctan(1/3)/3)

• ∛(3 + i ) = ⁶√10 exp(i (arctan(1/3)/3 + 2π/3))

• ∛(3 + i ) = ⁶√10 exp(i (arctan(1/3)/6 + 4π/3))

COLOR THEME
Q ZOOM
7. The dimensions in the drawing below are given in units.
3V10
х
36
Р
118
S
R
90
18V5
What is the value of x?
45
CLEAR ALL

Answers

Given:

In triangle PQR, QS⊥PR, QS=x, PR=90 and RS=18.

To find:

The value of x.

Solution:

According to right triangle altitude theorem (geometric mean theorem), if an altitude of a right triangle divides the base into two segments then the square of altitude is equal to the product segments of the base.

Using right triangle altitude theorem, we get

[tex]\dfrac{QS}{PS}=\dfrac{RS}{QS}[/tex]

[tex]QS^2=RS\times PS[/tex]

[tex]x^2=18\times (90-18)[/tex]

[tex]x^2=18\times 72[/tex]

[tex]x^2=1296[/tex]

Taking square root on both sides, we get

[tex]x=\sqrt{1296}[/tex]

[tex]x=36[/tex]

Therefore, the correct option is B.

Which statement is true about the graphed function?
O Fx) < 0 over the interval (-0, 4)
OFx) < 0 over the interval (-0, -3)
O F(x) > 0 over the interval (-00, -3)
O F(x) > 0 over the interval (-0, 4)

Answers

Answer:

D is the answer to the question

What is the slope of the line containing (-3, 1) and (1, -2

Answers

Answer:

-3 on 4

Step-by-step explanation:

slop is given by change in y coordinate over change in x coordinate

It will be -3, 4 !!!

help me solve this question its coming in my next math test and i cant solve it!​

Answers

Answer: sorry I can't help you

Step-by-step explanation: because the picture won't show up.

Which of the following is not a property of a rectangle

Answers

Answer:

I would like to say A because Diagonals of a rectangle Bisect each other at a right angle.

This seems likely

Please Mark as Brainliest

Hope this Helps

Answer:

I think C would be the best answer.

Step-by-step explanation:

Sorry if it was not right ):

natally needs to cut a pice of paper so that she has exactly 1/3 of it.if the length of the paper is 11 inches, how long will be the paper if she cuts . help me please

Answers

Answer:

3 [tex]\frac{2}{3}[/tex] inches long

Step-by-step explanation:

Find how long the paper will be by dividing 11 by 3. This will find how much 1/3 of the 11 inch paper is.

11/3

= 3 [tex]\frac{2}{3}[/tex]

So, if she cuts the paper, the paper will be 3 [tex]\frac{2}{3}[/tex] inches long

PLEASE ANSWER ASAP FOR BRAINLEST!!!!!!!!!!!!!!!!!!

Answers

Answer:

50 %

Step-by-step explanation:

Please mark me as the brainlist

Answer:

answer is 37.5%

Step-by-step explanation:

the only prime numbers after adding are

4+9= 13

4+7=11

3+8=11

place the sum # of prime numbers over the total some of numbers

3÷8= 0.375

now multiply by 100 to give you in percentage

0.375 x 100% = 37.5%


A weatherman predicted 8 inches of snow would fall last night.
Instead, 20 inches of snow actually fell.
What is the percent error of the prediction?
about 12%
about 40%
about 60%
about 150%

Answers

Answer:

The correct answer is - about 60%

Step-by-step explanation:

Given:

predicted value: 8 inches

measured value - 20 inches

formula:

Prediction percentage error is the error or differences between the prediction made and the observed or actual value. Calculation of the percentage error by the following equation.

prediction percentage error = (measured value - predicted value)*100/measured value

solution:

putting value in the formula

= (20-8)*100/20

= 12*5

= 60 %

how are they the same

Answers

Answer:

Step-by-step explanation:

The ways that they are the same are.

The slope is the same.Since the slope is the same for both of them, there is no possible solution meant for both of them. The slope is 1/4They are both written in slope y intercept form.The only difference is the y intercept is different. y intercept for y = 1/4 x is 0,0y intercept for y = 1/4 x is 0,5

Suppose we are interested in bidding on a piece of land and we know one other bidder isinterested. The seller announced that the highest bid in excess of $10,000 will be accepted.Assume that the competitor’s bid x is a random variable that is uniformly distributedbetween $10,000 and $15,000.a. Suppose you bid $12,000. What is the probability that your bid will be accepted?b. Suppose you bid $14,000. What is the probability that your bid will be accepted?c. What amount should you bid to maximize the probability that you get theproperty?d. Suppose you know someone who is willing to pay you $16,000 for the property. Wouldyou consider bidding less than the amount in part (c)? Why or why not?

Answers

Answer:

a. 0.4 = 40% probability that your bid will be accepted

b. 0.8 = 80% probability that your bid will be accepted.

c. $15,000.

d. Bidding $15,000 guarantees you a profit of $1,000, while trying to bid less than $15,000, you can end up without a profit, thus, you should not consider bidding less than the amount in part (c).

Step-by-step explanation:

Uniform probability distribution:

An uniform distribution has two bounds, a and b.

The probability of finding a value of at lower than x is:

[tex]P(X < x) = \frac{x - a}{b - a}[/tex]

Assume that the competitor’s bid x is a random variable that is uniformly distributed between $10,000 and $15,000.

This means that [tex]a = 10, b = 15[/tex], considering the measures in thousands of dollars.

a. Suppose you bid $12,000. What is the probability that your bid will be accepted?

Probability that the other bidder's bid is less than 12000(X < 12). So

[tex]P(X < 12) = \frac{12 - 10}{15 - 10} = \frac{2}{5} = 0.4[/tex]

0.4 = 40% probability that your bid will be accepted.

b. Suppose you bid $14,000. What is the probability that your bid will be accepted?

Probability that the other bidder's bid is less than 14000(X < 14). So

[tex]P(X < 14) = \frac{14 - 10}{15 - 10} = \frac{4}{5} = 0.8[/tex]

0.8 = 80% probability that your bid will be accepted.

c. What amount should you bid to maximize the probability that you get the property?

The upper bound of the uniform distribution, that is, $15,000.

d. Suppose you know someone who is willing to pay you $16,000 for the property. Would you consider bidding less than the amount in part (c)? Why or why not?

Bidding $15,000 guarantees you a profit of $1,000, while trying to bid less than $15,000, you can end up without a profit, thus, you should not consider bidding less than the amount in part (c).

Before simplifying, how many terms are there in the expression -2x+8 -6y + 8x + 11?
5
6
3
4


Answers

Answer:

There are 5 terms in the given expression.

a) Two thirds of the class are freshman

Is it parameter or statistic

Answers

Answer:

parameter

Step-by-step explanation:

Parameter is the answer

I kinda need help!!!!!!!!!!!!!!!!

Answers

Answer:

15+15+15=45

Step-by-step explanation:

8+8+8=24

4+4+4=12

4+15*8=

124

Answer:

Answers is hear that.

4+15+8=27

A rectangle is to be inscribed in an isosceles right triangle in such a way that one vertex of the rectangle is the intersection point of the legs of the triangle and the opposite vertex lies on the hypotenuse. Find the largest area (in cm 2 ) of the rectangle and its dimensions (in cm) given that the two equal legs of the triangle have length 4.

Answers

Answer:

x  =  2  cm

y  = 2  cm

A(max) =  4 cm²

Step-by-step explanation: See Annex

The right isosceles triangle has two 45° angles and the right angle.

tan 45°  =  1  =  x / 4 - y        or     x  =  4  -  y     y  =  4  -  x

A(r)  =  x* y

Area of the rectangle as a function of x

A(x)  =  x  *  (  4  -  x )       A(x)  =  4*x  -  x²

Tacking derivatives on both sides of the equation:

A´(x)  =  4 - 2*x             A´(x)  =  0            4   -  2*x  =  0

2*x  =  4

x  =  2  cm

And  y  =  4  - 2  =  2  cm

The rectangle of maximum area result to be a square of side 2 cm

A(max)  = 2*2  =  4 cm²

To find out if A(x) has a maximum in the point  x  =  2

We get the second derivative

A´´(x)  =  -2           A´´(x)  <  0   then A(x) has a maximum at  x = 2

The maximum area of the rectangle inscribe in the right isosceles triangle

is [tex]\rm \bold{4\;cm^2}[/tex]

Given that a rectangle is to be inscribed in an isosceles triangle

As per the attached figure we can write the following

Let ΔABC  represents the isosceles triangle from the given data in the question we can write the following observations

Length of AB = 4

Length of BC = 4

∠ACB = 45°

∠BAC = 45°

Length of AC = [tex]\rm 4\sqrt2[/tex]

We have to consider a rectangle within the isosceles triangle such that  

area of rectangle is maximum

The area of rectangle is given by equation (1)

[tex]\rm Area \; of \; rectangle = Length \times Breadth ........(1) \\[/tex]

Let the area of rectangle inscribed by the isosceles triangle is " A"

Let the lenght of the rectangle be "x "

Let the width of the rectangle be " y "

So according to equation (1) the area of rectangle inscribed into the triangle is given by

[tex]\rm A = x \times y.......(2)[/tex]

In ΔAED we can write

[tex]\int\limits^a_b {x} \, dx \rm tan 45= \dfrac{Perpendicular}{Base } = \dfrac{4-y}{x} \\1 = \dfrac{4-y}{x} \\ x = 4-y .......(3) \\[/tex]

From equation (2) and (3) we get

[tex]\rm A = (4-y)(y) \\A = 4y -y^2......(4)[/tex]

On differentiating the given function with respect to y we get

[tex]\rm A' = 4- 2y \\[/tex]

On putting A' =0 we get

[tex]\rm 4-2y =0 \\2y = 4\\y = 2[/tex]

A'' is negative hence it is the maximum value of y

So we can conclude that the maximum value of y is 2

So from equation (4) the maximum area is [tex]\rm 2\times 2 = 4\; cm^2[/tex]

The maximum area of the rectangle inscribe in the right isosceles triangle

is [tex]\rm \bold{4\;cm^2}[/tex]

For more information please refer to the link given below

https://brainly.com/question/541676

Help with quadratic formula

Answers

Ok, but what is the question?

Answer:

The format for any quadratic equation is;

ax² + bx + c

And the quadratic formula is written as;

[tex]\frac{-b+-\sqrt{b^2 -4ac} }{2a}[/tex]

Hope this helps!

Ali surveyed his classmates to see who liked tennis. In all, 32 males liked tennis, and 6 males did not like tennis; 13 females liked tennis, and 4 females did not like tennis. How many females did Ali survey

Answers

Answer:

Ali surveyed 17 females.

Step-by-step explanation:

From Ali's survey:

13 females liked tennis, while 4 did not.

This means that Ali's surveyed 13 + 4 = 17 females.

How many females did Ali survey?

Ali surveyed 17 females.

A bicycle wheel has a diameter of 26 inches. What is the circumference?

Answers

Answer:

81.64 Inches

Step-by-step explanation:

Given :

Diameter of wheel =26 inches

Radius of wheel (r)=26/2=13 inches

Circumference (C) of wheel can be calculated as :

C=  2πr

C=6.28(13)

C=81.64 Inches

Calculamos el volumen (V) de un prisma cuadrangular expresado en unidades cúbicas. Respondemos: ¿cómo se calcula?........................... entonces, la expresión para calcular el volumen es: V = ……………………

Answers

Answer:

a) El procedimiento queda resumido en lo que sigue:

1) Se halla el área de la base del cuadrilátero.

2) Se multiplica el resultado anterior por la longitud del prisma.

b) La expresión para calcular el volumen del prisma ([tex]V[/tex]), en unidades cúbicas, es:

[tex]V = w\cdot h \cdot l[/tex]

Donde:

[tex]w[/tex] - Ancho de la base, en unidades.

[tex]h[/tex] - Altura de la base, en unidades.

[tex]l[/tex] - Longitud del prisma, en unidades.

Step-by-step explanation:

a) Dimensionalmente hablando, la unidad de volumen es igual al cubo de la unidad de longitud. Se explica a continuación los pasos necesarios para el cálculo de prisma cuadrangular:

1) Se halla el área de la base del cuadrilátero.

2) Se multiplica el resultado anterior por la longitud del prisma.

b) En consecuencia, se deriva la expresión para el volumen del prisma:

[tex]V = A\cdot l[/tex] (1)

Donde:

[tex]A[/tex] - Área de la base del prisma, en unidades cuadradas.

[tex]l[/tex] - Longitud del prisma, en unidades.

[tex]V = w\cdot h \cdot l[/tex] (2)

Donde:

[tex]w[/tex] - Ancho de la base, en unidades.

[tex]h[/tex] - Altura de la base, en unidades.

PLS HELP !!!! WILL MARK BRAINLIEST !!

Answers

Answer: Choice C) no solution

=============================================================

Explanation:

The first equation says that y and 3x-4 are the same thing.

So we can replace every copy of "y" in the second equation with "3x-4" like so

9x - 3y = 14

9x - 3(3x-4) = 14

9x - 9x + 12 = 14

0x + 12 = 14

0 + 12 = 14

12 = 14

The last equation is false because we get different values on either side. Because the last equation is false, it means the equation 9x-3y = 14 is false when y = 3x-4.

In short, the overall systems of equations is inconsistent and that leads to no solution (choice C).

If you were to graph each equation, you should find that both lines are parallel. Parallel lines never intersect. The solution is the intersection point of any graphed system.

Answer:

there is no solution to this problem

Step-by-step explanation:

I hope this helps

Please help quickly I really don’t understand this question and i’m freaking out.

Answers

Answer:

[tex] \frac{4 \sqrt{3x} }{9} [/tex]

Step-by-step explanation:

[tex] \frac{4 \sqrt{ {21x}^{2} } }{3 \sqrt{63x} } [/tex][tex] \frac{4 \sqrt{ \frac{21 {x}^{2} }{63x} } }{3} [/tex][tex] \frac{4 \sqrt{ \frac{x}{3} } }{3} [/tex][tex] \frac{4 \sqrt{ \frac{x \times 3}{3} } }{3} [/tex][tex] \frac{\frac{4 \sqrt{3x} }{3} }{3} [/tex][tex] \frac{4 \sqrt{3x} }{3} \times \frac{1}{3} [/tex][tex] \frac{4 \sqrt{3x} }{9} [/tex]Hope it is helpful...

Part A

Will be marked brainliest

Monique’s group will design a float for the I love my city parade. They plan to highlight the city’s train system by making a float of the MetroPass.

*SEE PICTURES FOR ACTUAL MEASURES*

The actual MetroPass is 2.1 inches wide and 3.4 inches long. Moniques group wants the MetroPass float to be 74.8 ft long. Her group needs to find the width of the MetroPass float.

How wide, in feet, will the MetroPass float be? Round you answer to the nearest tenth. Show your work.

*WILL BE MARKED BRAINLIEST*

Answers

Answer:

74.8 ft÷3.4 inches=264

2.1 in x 264=46.2 inches so the width should be 46.2 inches

Which table represents a linear function?
c
2
0
1
2
y
0
-1
4
-9
0
1
2
3
y
3
1
1
-3
2
0
y
0
1
4
9
y
1
3
0
1
2
3
1
--3
2
3
-1

Answers

Answer:

Table C is the answer

Step-by-step explanation:

Bernardo woke up one morning and saw frost on the windows. It was -15°C outside. Later in the day, the frost melted. At 3 p.m. it was 14°C outside. Which expression represents the difference between the two temperatures?

Answers

Answer:

something like: 14 - (-15)

Step-by-step explanation:

A debt of $9000 is to be amortized with 4 equal semiannual payments. If the interest rate is 7%, compounded semiannually, what is the size of each payment?

Answers

Answer:

Instalment = Loan / PVAF (r%, n)

Where r is int rate per six months & n is no. of half years

= $ 9000 / PVAF (5% , 4)

= $ 9000/ 3.5460

= $ 2538.11

Pls comment, if any further assistance is required.

Step-by-step explanation:

(3+8i) (3+8i) multiply

Answers

Answer:-55+48i

Step-by-step explanation:

Answer:

Step-by-step explanation:

73

What is the horizontal distance between (−5/3, -2) and (4, -2)?


ANSWER QUICK GIVING BRAIN LEST

Answers

Answer:

-2?

Step-by-step explanation:

use y2-y1 over x2-x1 to find the answer. So you would do -2--5/3 over 4--2. The two negitives turn into a positive. SO it is now -2+5/3 over 4+2. -2+5/3= -1/3 over 4+2=6 it is now -1/3/6 which equals -2. So I think the answer is -2. I do not know if you should trust me on this but I tried.

Helpppppppppppppp:))))

Answers

Given:

Principal value = $1500

Rate of interest = 7% per annum compounded daily

Time = 2 years.

To find:

The amount after 2 years.

Solution:

Formula for amount:

[tex]A=P\left(1+\dfrac{r}{n}\right)^{nt}[/tex]

Where, P is principal, r is the rate of interest in decimals, n is the number of time interest compounded in an year and t is the number of years.

We know that 1 year is equal to 365 days and the interest compounded daily. So, n=365.

Substituting [tex]P=1500,\ r=0.07,\ n=365,\ t=2[/tex] in the above formula, we get

[tex]A=1500\left(1+\dfrac{0.07}{365}\right)^{365(2)}[/tex]

[tex]A=1500\left(\dfrac{365+0.07}{365}\right)^{730}[/tex]

[tex]A=1500\left(\dfrac{365.07}{365}\right)^{730}[/tex]

Using calculator, we get

[tex]A\approx 1725.39[/tex]

The amount after two years is $1,725.39. Therefore, the correct option is (c).

Other Questions
Diameter of the BalloonBefore HeatAfter Heat6 centimeters 10 centimetersWhich conclusion is supported by the evidence?A. Air expands when heated.B. Air is warmer when heated.C. The balloon will pop when heated.D. The balloon contracts when heated. Please Help me... You live in ancient West Africa. Your family is traveling to East Africa. Write a paragraph describing what you might experience when you arrive in East Africa. Tell us about the people, land, and weather. A Middle School takes a poll to find out what students want to eat at lunch. 70 students from 8th Grade are randomly chosen to answer the poll questions. Identify the population and sample? how many children did okonkwo have Calculate the number of representative particles of 0.0391 mol Neon gas.i need help ASAP ! One fourth of the sum of a number and two is three. Help meeeeeeeeeeeeeeeeeeeee Given right triangle ABC, what is the value of tan(A)? what are the three main particles of soil are? How do these shape and influence the type of soil we observe and its uses? In Korematsu v. US (1944), the Supreme Court ruled that in a time of great emergency and peril, the internment of Japanese Americans was As you read the short story, record any details that relate to the topic of American identity. busco suggar para mi amiga de 20 a 30 xd A quarter back completes 23% of his passes. We want to observe this quarterback during one game to see how many pass attempts he makes before completing one pass. What is the probability that the quarterback throws exactly 6 incomplete passes before he has a completion What is the congruence correspondence, if any, that will prove the given triangles congruent? A rectangular dog pen is constructed using a barn wall as one side and 64 m of fencing for the other three sides what is the maximum area of the dog pen pls help me solve this please show how you got the answer Which passage most clearly contains imagery?O A. "Don't say another word!" Gina shouted.B. The odor of rotten eggs wafted from within.C. He suspected. He knew. He saw. He smiled.D. First, call our hotline. Next, work out a plan.SUBM A child on a tricycle is moving at a speed of 1.40 m/s at the start of a 2.25 m high and 12.4 m long incline. The total mass is 48.0 kg, air resistance and rolling resistance can be modeled as a constant friction force of 41.0 N, and the speed at the lower end of the incline is 6.50 m/s. Determine the work done (in J) by the child as the tricycle travels down the incline. I need to know the area in square centimeters of sector ABC? Copy the diagram. add a voltmeter to show how you would measure the voltage of the cell